Using symmetry of action to find the constant of motion

Click For Summary
The discussion focuses on using symmetry transformations to find constants of motion in a variational problem related to Noether's theorem. The transformation applied, e^{i \alpha }\psi, indicates a symmetry of the action, prompting a need to connect this with the constant of motion. Participants suggest studying the Noether procedure further and applying it to the specific example at hand. They also highlight the importance of handling boundary terms correctly during the variational procedure, as relaxing assumptions about these terms can reveal additional insights into conserved quantities. Understanding and applying these concepts is crucial for solving the problem effectively.
Pushoam
Messages
961
Reaction score
53

Homework Statement


upload_2019-2-12_1-15-44.png
upload_2019-2-12_1-16-7.png


Homework Equations

The Attempt at a Solution


I need help in solving second part of this question.
I put ## e^{i \alpha }\psi ## instead of ##\psi ## and got to see that the integrand doesn't change which means the given transformation is a symmetry of the given action. But how to use this information to find out constant of motion?
I have seen a lot of pdf which talk of the transformation ## \psi ## going to ## \psi + \epsilon \eta (x) ##. Since, the transformation given in this problem is different. I do not know with what I should start.
 

Attachments

  • upload_2019-2-12_1-15-44.png
    upload_2019-2-12_1-15-44.png
    31.9 KB · Views: 629
  • upload_2019-2-12_1-16-7.png
    upload_2019-2-12_1-16-7.png
    11.2 KB · Views: 645
  • Like
Likes Delta2
Physics news on Phys.org
Two big hints here... the phrase "treating it as a variational problem in t" and "show that the Noether procedure leads to..." You need to study up on that procedure. I suggest you come back here with more specific questions about that procedure if you're stuck.

I think basically your instructor wants you to show that you understand Noether's proof of her celebrated theorem by applying it in this specific example. Also note that when you carry out the variational procedure (the one usually executed to derive the Euler Lagrange equations of motion) you will get some boundary terms which you need not assume go away. We usually assert \delta t, \delta x are zero on the boundary region. I vaguely recall relaxing this assumption leads to some interesting additional terms having to do with fluxes of conserved quantities across the boundary. But let me emphasize the "vaguely" in that sentence.
 
  • Like
Likes Pushoam
Question: A clock's minute hand has length 4 and its hour hand has length 3. What is the distance between the tips at the moment when it is increasing most rapidly?(Putnam Exam Question) Answer: Making assumption that both the hands moves at constant angular velocities, the answer is ## \sqrt{7} .## But don't you think this assumption is somewhat doubtful and wrong?

Similar threads

  • · Replies 3 ·
Replies
3
Views
2K
  • · Replies 3 ·
Replies
3
Views
2K
  • · Replies 8 ·
Replies
8
Views
2K
  • · Replies 22 ·
Replies
22
Views
2K
  • · Replies 7 ·
Replies
7
Views
3K
  • · Replies 5 ·
Replies
5
Views
1K
  • · Replies 6 ·
Replies
6
Views
1K
Replies
5
Views
2K
Replies
18
Views
4K
Replies
56
Views
5K